[obm-l] Soluções IME 2006

2005-10-26 Por tôpico yurigomes
Oi pessoal, Quem estiver interessado nas soluções da prova de matemática do IME 2006 dê uma olhadinha em www.c7s.com.br Tá bem legal! Abraços, Yuri Até mais, Yuri = Instruções para entrar na lista, sair da lista e

[obm-l] Resultado - XX Ibero

2005-09-30 Por tôpico yurigomes
Oi pessoal, Estou enviando agora as pontuaccoes por problemas e os resultados. Bem, conseguimos novamente o máximo que podíamos: 4 ouros! Um deles foi puntaje perfecto, o Gabriel. As pontuaccoes foram as seguintes: 1 2 34 56 TOTAL BRA 1 (Thiago)

[obm-l] Primeiro dia - XX Ibero

2005-09-28 Por tôpico yurigomes
Oi pessoal, Estou trazendo notícias da Ibero. Nossa viagem foi um pouco complicada. Qdo chegamos no aeroporto de Guarulhos descobrimos que nosso voo, que partiu de RJ, teria uma troca para uma aeronave menor, e isso causou um overbooking enorme. Entao todos que partiam de SP, eu, Gabriel,

[obm-l] Segundo dia - XX Ibero

2005-09-28 Por tôpico yurigomes
Estou enviando agora os problemas do segundo dia. Ainda nao sei como os meninos foram pq desde depois do almocco eles estao na prova por equipes e nem chegaram a voltar pro hotel. O problema 5 é do Brasil (meu e do Davi). =)) PROBLEMA 4 Dados dois inteiros positivos a e b, denota-se por (a mod

[obm-l] Cone Sul 2005 - prova e resultados

2005-05-28 Por tôpico yurigomes
Oi pessoal, Notícias e resultados da Cone Sul... Os problemas estão no final do e-mail. 1o e-mail: Ai vai o primeiro dia da Cone Sul aqui na Bolívia. Foi um dia fácil, e espero que os garotos tenham ido bem, ainda não vi as provas deles. O problema 1 é da Bolívia, o 2 é nosso (Cícero), e o 3

[obm-l] Re: [obm-l] [OFF] Sir Michael Atiyah no Brasil

2005-03-17 Por tôpico yurigomes
Só corrigindo, ele foi Medalha Fields em 66, até porque ele já tem mais de 70 anos!! -- Mensagem original -- Para quem se interessa por matemática...Sir Michael Atiyah, um dos maiores matemáticos da atualidade, detentor da melhada Fields de 96 e do prêmio Abel de 04, fará uma palestra gratuita

[obm-l] Re: [obm-l] Variáveis complexas

2005-01-02 Por tôpico yurigomes
Oi Tertuliano, 1) Suponha que f(z) =! 0, para todo z em U. Considere g = 1/f. Então g tem um máximo local, a dizer z = a, e portanto deve ser constante. 2) Vamos mostrar que f^(n+1)(z) = 0, para todo z em U. De fato, tome r max{R, |z|}. Então pela fórmula integral de Cauchy temos: f^(n+1)(z)

[obm-l] =?iso-8859-1?Q?Re=3A=20=5Bobm=2Dl=5D=20questao=20do=20ITA=20furada?=

2004-12-16 Por tôpico yurigomes
Oi pessoal, Pelo visto a prova do ITA tá dando muita controvérsia. A solução da questão abaixo (questão 30) está com um pequeno erro, uma vez que a=1 também gera 3 soluções, que são 0, 1 e 2. Divulgamos a prova com soluções em www.c7s.com.br. Acho que muitas dessas controvérsias foram

[obm-l] Solucoes da IMC

2004-07-27 Por tôpico yurigomes
Oi Marcio e pessoal da lista, Estamos aguardando a recorrecao que serah amanha, visto que acho que terah muita mudanca depois disso. Qto aas suas solucoes: PRIMEIRO DIA Prob 1: todo mundo fez considerando subconjuntos contidos nos intervalos [1/(k+1), 1/k] e o reciproco negativo. Cada

[obm-l] Prova da IMC - 1o. dia

2004-07-25 Por tôpico yurigomes
Oi pessoal, O Thiago Barros teve a paciencia de redigir os enunciados dos problemas do 1o. dia da IMC - 2004 para que vcs pudessem ver e pediu que eu encaminhasse-as para a lista. Os enunciados estao abaixo. Ate mais, Yuri 1) Let S be an infinite set of real numbers such that |s_1 + s_2

[obm-l] Re: [obm-l] Prova da IMC - 1o. dia (Correcao)

2004-07-25 Por tôpico yurigomes
Tem soh um errinho pequeno porem decisivo no enunciado do prob 1. Embaixo estah ajeitado. -- Mensagem original -- Oi pessoal, O Thiago Barros teve a paciencia de redigir os enunciados dos problemas do 1o. dia da IMC - 2004 para que vcs pudessem ver e pediu que eu encaminhasse-as para a

[obm-l] Prova da IMC - 1o. dia (correcao)

2004-07-25 Por tôpico yurigomes
Bem, num sei se a correcao que fiz no ultimo email vai chegar, entaum estou novamente corrigindo o enunciado do problema 1. 1) Let S be an infinite set of real numbers such that |s_1 + s_2 + ... + s_k| 1 for every finite subset {s_1,s_2,...,s_k} of S. Show that S is countable. 2)Let P(x) =

[obm-l] Re: [obm-l] OLIMPÍADA CEARENSE!

2004-03-05 Por tôpico yurigomes
Eu fiz essa prova! Eu acho Observe o seguinte: os números 2, 4, 8, 16 e 32 devem estar em caixas distintas, pois senão a condição do mdc não seria satisfeita. Então temos um total de caixas maior ou igual a 5. Agora basta mostrar um exemplo com 5 caixas. Acho que colocando os números 2^i,

[obm-l] Re: [obm-l] Mais Ramsey

2004-03-04 Por tôpico yurigomes
Prove que se pintarmos cada aresta de um grafo completo de ordem 6 com uma dentre duas cores, este grafo conterah um subgrafo monocromatico de ordem 3 (um triangulo, por assim dizer). Na verdade, é possível mostrar que existem DOIS triângulos monocromáticos! Até mais, Yuri []'s, Yuri ICQ:

[obm-l] Re: [obm-l] Teoria dos números....

2004-02-03 Por tôpico yurigomes
Oi Crom, Tem algo errado nessa fração. Tome n=5. Então, se não interpretei errado, a fração vale: 4.(4! + 1)/(5.7) = 4.25/(5.7) = 20/7, que não é inteiro... Ateh mais, Yuri -- Mensagem original -- 1)Seja n=2 um número ineiro. Prove que n e n+2 são ambos primos se e somente se

[obm-l] Re: [obm-l] Re: [obm-l] Polinomio divisivel por m

2003-11-23 Por tôpico yurigomes
Pronto! Soh um detalhe. O argumento que fiz abaixo mostra que existe n tal que f(n)=0 (mod p) qdo p é diferente de 13 e 17. Para completar essa parte, basta observar que (17/13) = (4/13) = (2/13)^2 = 1. e que pela lei de reciprocidade quadrática: (13/17)= (-1)^(6x8).(17/13) = 1. Para o caso

[obm-l] Re: [obm-l] Grupo Abeliano

2003-10-31 Por tôpico yurigomes
Oi Eduardo, Eu acho que vc se confundiu na definição de H. Do jeito que vc colocou, H teria n^n elementos. Eu acho que vc estava querendo dizer GxG, estou certo? Nesse caso, H teria n^2 elementos... Ateh mais, Yuri -- Mensagem original -- Oi, Duda: Infelizmente, tenho que discordar. H_(n+1)

[obm-l] Re: [obm-l] obm - U

2003-10-21 Por tôpico yurigomes
Vc pode fazer essa desigualdade por Cauchy: observe (SOMA{(sr(p_i^3))^2}).(SOMA{((sr(p_i))^2} = (SOMA{sr(p_i^3).sr(p_i})^2 Mas o segundo fator do lado esquerdo é igual a SOMA(p_i)=1, e o resultado segue. Outra maneira seria observar que SOMA{p_i^3) = SOMA{p_i^3).SOMA{p_i) =

[obm-l] Re: [obm-l] Algebra linear : Wronkisano e indicacao de livro

2003-09-20 Por tôpico yurigomes
Oi Niski, Eu acho que dá pra fazer pelo wronskiano mesmo. Vamos lá: Derivando e^(a(j).x) i vezes, ficamos com a(j)^i.e^(a(j).x). Logo: W= W(e^(a(1).x),...,e^(a(n).x))= det((a(j)^(i-1).e^(a(j).x)), onde i representa a linha e j a coluna. Agora observe que todos os elementos da coluna j têm

[obm-l] Re: imo

2003-08-17 Por tôpico yurigomes
Se não me engano, o problema C5 é do Gugu. Ele caiu uma semana atrás no teste pra Ibero desse ano. Ateh mais, Yuri []'s, Yuri ICQ: 64992515 -- Use o melhor sistema de busca da Internet Radar UOL - http://www.radaruol.com.br

[no subject]

2003-08-17 Por tôpico yurigomes
Se não me engano, o problema C5 é do Gugu. Ele caiu uma semana atrás no teste pra Ibero desse ano. Ateh mais, Yuri []'s, Yuri ICQ: 64992515 -- Use o melhor sistema de busca da Internet Radar UOL - http://www.radaruol.com.br

[obm-l] Re: [obm-l] Teoria dos Números

2003-08-16 Por tôpico yurigomes
Oi Claudio, Eu não entendi pq vc considerou polinômios para provar a última passagem, jah que a está fixo. Ou seja, vc tem que a^n - 1 divide a^Phi(a^n - 1) - 1 e não que x^n-1 divide x^Phi(a^n - 1) - 1 para todo x. Se eu tiver falado alguma besteira, me avisem! Ateh mais, Yuri --

[obm-l] Re: [obm-l] Re: [obm-l] Teoria dos Números

2003-08-16 Por tôpico yurigomes
Exatamente! Vejam a minha msg anterior... -- Mensagem original -- Oi, Yuri: O que eu provei foi o seguinte: m divide n == p(x) = x^m - 1 divide q(x) = x^n - 1 (na verdade, eu provei soh a volta, mas a ida eh imediata) Em particular, com um inteiro a fixo: m divide n == p(a) divide q(a). Ou

[obm-l] Re: [obm-l] Re: [obm-l] Teoria dos Números

2003-08-16 Por tôpico yurigomes
Opa! Na verdade vale uma coisa mais geral! a^n- 1 divide a^m- 1 = n divide m. Dessa forma, tirei a minha dúvida. Além disso, a prova do Cláudio prova também a afirmação acima. Ateh mais, Yuri -- Mensagem original -- Oi Claudio, Eu não entendi pq vc considerou polinômios para provar a

[obm-l] Re: [obm-l] Problemas em Aberto - Algarismos

2003-08-14 Por tôpico yurigomes
Esse segundo problema caiu na OBM 2000, numa versão mais fácil. Acho que foi essa versão a que vc resolveu, jah que ele dizia que as duas potências têm que ter o mesmo número de algarismos, de modo que os zeros não modificavam a quantidade de algarismos. Ateh mais, Yuri -- Mensagem

[obm-l] Re: [obm-l] Re: [obm-l] Cochilo na aula de algebra

2003-08-11 Por tôpico yurigomes
Oi Henrique, A motivação disso foi o enunciado dizer que todas as raízes são reais e positivas. Nada melhor do que média nesse caso!! Se não houvesse igualdade, nada garantiria que as raízes fossem todas iguais a 2. De fato, poderiam haver várias possibilidades para o conjunto das dez

[obm-l] Re: [obm-l] Cochilo na aula de algebra

2003-08-10 Por tôpico yurigomes
Todas as raízes são iguais a 2. De fato, se as raízes são x_1, x_2,..., x_10, então pelas relações de Girard temos: x_1 + x_2 + ... + x_10= 20 x_1.x_2...x_10= 1024 Como as raízes são reais positivas, podemos usar MA = MG: (x_1 + x_2 + ... + x_10)/10 = (x_1.x_2...x_10)^(1/10) =

[obm-l] Re: [obm-l] Problemas em Aberto - Algarismos

2003-08-10 Por tôpico yurigomes
Essa primeira questão pode conte repetições, como por exemplo 33600??? -- Mensagem original -- Caros colegas: Aqui vao dois problemas que ainda estao em aberto na lista. O primeiro foi enviado pelo Duda Stabel. O segundo eh da olimpiada iraniana, se nao me engano. 1) Determinar o conjunto de

[obm-l] IMC - problema 2 (2o dia)

2003-08-01 Por tôpico yurigomes
Segue abaixo a solução do problema 2 do 2o dia. Vou deixar um espaço pra quem quiser tentar! 02) Calcule o seguinte limite 2x / lim| (sin t)^m/t^n dt (m,n naturais) x-0+ / x . . . . . . . . . . . . . . . . Pronto! Vamos dividir em três

[obm-l] Re: [obm-l] IMC - problema 1

2003-07-31 Por tôpico yurigomes
Oi pessoal, segue abaixo a minha solução do problema 1 do primeiro dia da IMC. Ah, parabéns aa equipe brasileira!! Foi um ótimo resultado para uma primeira participação nessa competição!!! 1)a) Seja a1, a2, ... , an, ... uma sequencia de numeros reais tais que a1=1 e a(n+1)3/2*an para todo

[obm-l] Re: [obm-l] Paradoxo

2003-07-31 Por tôpico yurigomes
O que acontece nessa figura é que os dois coeficientes angulares das retas que são coladas não são os mesmos, de modo que fica sobrando um espaço na colagem, equivalente a uma quadrado de lado um, exatamente a diferença entre as áreas das duas figuras. -- Mensagem original -- O Biagio

[obm-l] Re: [obm-l] IMC - problema 4

2003-07-30 Por tôpico yurigomes
Vou dar minha solução: eu considerei AUB=N partição. Sejam a, b tais que a.A= b.B. Podemos supor, WLOG, que 1 está em A. Então a está em B, de modo que existe d em B tal que a=b.d. Temos então que b|a, e ainda aA=db.A = d.A=B. Nosso problema se restringiu então a acharmos d natural tal

[obm-l] Re: [obm-l] IMC dia 2

2003-07-30 Por tôpico yurigomes
Consegui o item (a). Tou tentando o (b). Sem alguém puder ajudar.. 5. (a) Show that for each funtion f:QxQ - R there exists a fnction g:Q-R such that f(x,y)=g(x)+g(y) for all x,y in Q. (b) Find a function f:RxR - R for which there is no function g:R-R such that f(x,y) = g(x) + g(y) for all

[obm-l] Re: [obm-l] Re: [obm-l] IMC - problema 4

2003-07-30 Por tôpico yurigomes
Ops, me esqueci de falar que d1 (!!) A solução é então (a,b) tais que a!=b e mdc(a, b)=min{a,b} -- Mensagem original -- Vou dar minha solução: eu considerei AUB=N partição. Sejam a, b tais que a.A= b.B. Podemos supor, WLOG, que 1 está em A. Então a está em B, de modo que existe d em B

[obm-l] Re: [obm-l] Re: [obm-l] geometria

2003-07-25 Por tôpico yurigomes
É só trocar. É que na minha figura M e N ficaram acima de A, qdo a posição correta é abaixo. -- Mensagem original -- Ola pessoal, No enunciado foi dito que MB= 7 cm e NC= 4 cm, mas na resolucao eh dito que MB= 4 e NC = 7. Eh assim mesmo ? Em uma mensagem de 24/7/2003 23:44:20 Hora padrão

[obm-l] Re: [obm-l] Re: [obm-l] geometria 2

2003-07-25 Por tôpico yurigomes
Se naum me engano na notação adotada no problema, o valor de AX é sempre igual a p-a, onde p é o semiperímetro e a= BC, independente do triângulo. No caso em que o triângulo, os pontos A, X, I e o outro pto de tangência da circunferência inscrita a ABC formam um quadrado, e assim os lados são

[obm-l] Re: [obm-l] geometria

2003-07-24 Por tôpico yurigomes
Oi Rodrigo, Seja ABC=B e ACB=C. Então NCP= 90- C/2. Como NP//BC, temos CNP=C. Logo, NPC= 180- (C+ 90- C/2)= 90- C/2 = CNP isósceles = NP=NC=7 = MN+ MP= 7. De modo análogo, BMP= 180- B e MBP= B/2 = BPM= B/2 = BMP isósceles = MP= MB= 4. Logo, MN= 7- MP= 7- 4 = MN= 3. Ateh mais, Yuri --

[obm-l] Re: [obm-l] geometria 2

2003-07-24 Por tôpico yurigomes
Sejam a e b os comprimentos dos catetos, I o incentro de C1 e X o ponto de tangência de C1 com AC. Então o raio de C é igual a AX, e eh esse valor vale r= p- Hipotenusa= (a+b-Hipotenusa)/2= [a+b- sqr(a^2+b^2)]/2= k/2 - sqr(a^2+b^2)/2 O raio de C2 é a metade da hipotenusa: R= sqr(a^2+b^2)/2.

[obm-l] Re: [obm-l] Resultado do Brasil na IMO

2003-07-23 Por tôpico yurigomes
Oi João, Naum achei. Eh exatamente nesse link?? -- Mensagem original -- At 21:57 16/7/2003 -0300, you wrote: Os totais minimos para bronze, prata e ouro foram respectivamente 13, 19 e 29. Assim o resultado do nosso time foi o seguinte: 1 2 3 4 5 6

[obm-l] Re: [obm-l] sequências....

2003-07-21 Por tôpico yurigomes
Oi Crom, Aih vão as soluções: 1) Vamos mostrar por indução. Para n=1, temos a_1^3=a_1^2 = a_1=0 ou a_1=1.OK. Além disso, 1+ 8.a_1 é quadrado perfeito. Suponha por indução que a_1, ...a_(n-1) sejam inteiros e que 1+ 8(a_1+...+a_(n-1)).( Vc vai jah perceber pq essa ultima condição). Logo

[obm-l] Re: [obm-l] IMO - P1

2003-07-19 Por tôpico yurigomes
Oi Marcio, Soh hj eu li seu email, depois que eu tbm consegui fazer a questão. Tem apenas um detalhe que vc não observou: os t_i´s devem ser distintos, pq senão os dois conjuntos seriam iguais. Seguindo a sua notação, sendo D_i=(D+ t_i)U(t_i- D), temos |D_i|= 2.5050. O t_(i+1) deve ser

[obm-l] Re: [obm-l] Re: [obm-l] Combinatoria

2003-07-14 Por tôpico yurigomes
O principio da casa dos pombos (PCP), ou Principio de Dirichlet, na sua forma mais simples, diz que se vc tem n+1 bolas e quer distribuí-las em n gavetas, então algumas das gavetas deverá conter no minimo duas bolas. Isso eh bem intuitivo. Para provar isso, suponha por absurdo que não. Então

[obm-l] Re: [obm-l] Prova da IMO

2003-07-13 Por tôpico yurigomes
Na verdade ela jah deveria estar, pq o horario da prova aqui no Brasil foi ontem (sábado) aa noite. Hj aa noite serah realizado o segundo dia. Ateh agora ainda não achei. Se alguém conseguir, favor mandar para a lista. Ateh mais, Yuri -- Mensagem original -- Hoje será o primeiro dia de

[obm-l] Re: [obm-l] Combinatoria

2003-07-12 Por tôpico yurigomes
Oi Marcio, Se eu não me engano, esse problema tem no Problem Solving: Seja x_i= número de partidas jogadas até o dia i, inclusive. Como o enxadrista joga no minimo 1 partida por dia e no máximo 11x12= 132 no total, temos 1= a_1 a_2... a_77= 132. Some 20 na desigualdade: 21= a_1 +

[obm-l] Re: [obm-l] QUESTÕES INTERESSANTES

2003-07-09 Por tôpico yurigomes
Oi Frederico, Gostei das questões! =P (1)( = ) Suponha A e A^(-1) com entradas inteiras. Então detA e detA^(-1) são inteiros. Mas como detA.detA^(-1)= 1, devemos ter detA= +-1. ( = ) Ora, se A= (a b), então A^(-1)= 1/detA.(d -b), e assim (c d) (-c

[obm-l] Re: [obm-l] Problema de algum ano do IME

2003-06-08 Por tôpico yurigomes
Eh verdade. Eu coloquei o sinal errado. Eh menos mesmo. A indução eh pra ver que de dois em dois fatores, a expressão vai diminuindo de tamanho. Ateh mais, Yuri -- Mensagem original -- Yuri, como to estudando pro ITA, IME tb me interessa... comecei a ver sua resposta, teve uma passagem que

[obm-l] Re: [obm-l] Re: [obm-l] Problema de algum ano do IME(corrigindo)

2003-06-08 Por tôpico yurigomes
O Ariel percebeu um erro de sinal. Segue a correção abaixo. -- Mensagem original -- Oi Leo, Sempre que aparecerem expoentes que são potências de 2 consecutivas, um argumento que podemos fazer é ver o que acontece qdo multiplicamos a expressão por um valor que faz ela se reduzir a uma

[obm-l] Re: [obm-l] Re: [obm-l] Re: [obm-l] Problema de algum ano do IME(corrigindo)

2003-06-08 Por tôpico yurigomes
-- Mensagem original -- O Ariel percebeu um erro de sinal. Segue a correção abaixo. -- Mensagem original -- Oi Leo, Sempre que aparecerem expoentes que são potências de 2 consecutivas, um argumento que podemos fazer é ver o que acontece qdo multiplicamos a expressão por um valor que faz

[obm-l] Re: [obm-l] [E.M.] conjugado de complexos

2003-06-08 Por tôpico yurigomes
Basta vc observar que ~(v.w)=(~v).(~w). Para ver isso, chame v de a+bi e w de c+di e faça as contas: ~(v.w)= ~(ab-bd+ (ad+bc)i)= ab-bd-(ad+bc)i (~v).(~w)=(a-bi)(c-di)= ab-bd-(ad+bc)i Logo, aplicando isso n vezes, vc chega ao resultado. Ateh mais, Yuri -- Mensagem original -- bom, nao

[obm-l] Re: [obm-l] Re: [obm-l] f(f(x))_=_x^2_-_1996_é_impossível

2003-06-07 Por tôpico yurigomes
Oi Ricardo. Vc não pode fazer isso, já que não existe garantia de que f é derivável. Abraços, Yuri -- Mensagem original -- diretamente da lista... f(f(x))=x^2-1996..(1) derivando: f '(f(x)).f '(x)=2x ..(2) x^2-1996=(-x)^2-1996, entao: f(f(-x))=f(f(x))=x^2-1996,

[obm-l] Re: [obm-l] Problema de algum ano do IME

2003-06-07 Por tôpico yurigomes
Oi Leo, Smepre que aparecerem expoentes que são potências de 2 consecutivas, um argumento que podemos fazer é ver o que acontece qdo multiplicamos a expressão por um valor que faz ela se reduzir a uma expressão menor. No caso desse problema, seja

[obm-l] Re: [obm-l] Terorema de Feuerbach

2002-11-30 Por tôpico yurigomes
É interessante notar que, dado um triângulo ABC e H seu ortocentro, então C é ortocentro de ABH. Desse modo, o círculo dos nove pontos de ABH é o mesmo de ABC, e então esse círculo também é tangente ao incírculo e ex-incírculos de ABH. O mesmo valendo para ACH e BCH, concluímos que o círculo de

[obm-l] Re: [obm-l] IME 2003

2002-11-10 Por tôpico yurigomes
Essa era só perceber que 20 + 14sr(2)=(2+sr(2))^3. Logo, a expressão é igual a (2+sr(2))+ (2-sr(2))=4. -- Mensagem original -- Esta questão é da prova do IME que foi realizada nesta semana que passou. Alguém poderia me dar uma ajuda. Qual a melhor forma de resolver exercícios em que se tem

[obm-l] Re: [obm-l] questão 2 - IME 2003

2002-11-10 Por tôpico yurigomes
Um número é igual ao seu módulo sss ele é maior ou igual a 0. Logo log(12x^3 - 19x^2 +8x) = 0 12x^3 - 19x^2 +8x = 10^0 = 1 12x^3 - 19x^2 +8x - 1 = 0 (x-1)(x-1/3)(x-1/4) = 0 Analisando o sinal dessa função, deve ser 1/4 = x = 1/3 ou x = 1 -- Mensagem original --

[obm-l] Re: [obm-l] questão 4 - IME 2003

2002-11-10 Por tôpico yurigomes
tg(3a) = (tg(a)+tg(2a))/(1-tg(a).tg(2a)), donde tg(a) + tg(2a) = tg(3a).(1-tg(a).tg(2a)), e assim queremos tg(3a).(1-tg(a).tg(2a)) = 2.tg(3a) sss tg(3a)(1-tg(a).tg(2a) - 2) = 0 sss tg(3a)( tg(a).tg(2a) + 1) = 0 Caso i): tg(3a)=0 As soluções são a= 0 e a= pi/3 Caso ii): 1 +

[obm-l] Re: [obm-l] questão IME

2002-11-03 Por tôpico yurigomes
Sejam 2i+1,2i+3,...,2j+1 os termos da PA, com ij ( veja que i e j não obrigatoriamente são naturais). Entã a soma deles é (2i+1 + 2j+1)(j-i+1)/2, donde (j+1+i)(j+1-i)=7^3 Basta agora analisar os casos. Em cada um deles vc chegará num sistema e achará i e j. -- Mensagem original -- Alguem

[obm-l] Re: [obm-l] [IME96] Provar que eh periódica:

2002-10-27 Por tôpico yurigomes
Tente primeiro calcular f(x+2a) em função de f(x). Daí, vc chegará em f(x+2a)= 1/2 + sqr( 1/4 - f(x) + f(x)^2)= = 1/2 + sqr((1/2 - f(x))^2). Mas sabemos que f(x) é maior ou igual a 1/2, pois f(x)= 1/2 + sqr(algo), donde f(x)-1/2 é maior ou igual a zero. Logo, concluímos que f(x+2a)= 1/2 +

[obm-l] Re: [obm-l] E o Nivel Tres,ninguem faz nada??????

2002-10-26 Por tôpico yurigomes
Não. A condição de ser conjunto não permite repetição de elementos. Com repetição estamos falando de um MULTISET ( ou multiconjunto ). -- Mensagem original -- --- Eduardo Casagrande Stabel [EMAIL PROTECTED] escreveu: Olá. O Pessoal da Lista envelheceu junto com a Lista, por isso só se

[obm-l] Re: [obm-l] conesul

2002-07-31 Por tôpico yurigomes
Fala Carlos Tenta mostrar que o médio de DE, o incentro do ABC e o ponto T são colineares. Falow!! -- Mensagem original -- Alguem fez a 2 de geom. da conesul desse ano? empaquei nela... Obrigado, Carlos []'s, Yuri ICQ: 64992515 -- Use o melhor

[obm-l] Re: [obm-l] t. dos nºs

2002-06-11 Por tôpico yurigomes
Obs: o teorema anterior afirma que existem INTEIROS a e b. No problema p^2= a^2 + b^2 tem (0, p) como soluções inteiras. Se formos procurar soluções naturais, deveremos ter p|a^2 + b^2 . Suponha que p não divide a. Então seja c o inverso de a mod. p ( que existe, pois (a, p) ). Daí, p|(ac)^2

[obm-l] Re: [obm-l] Russas

2002-04-20 Por tôpico yurigomes
Para o problema 1 existe uma solução que prova ainda que 3(x+y)+1 e 4(x+y)+1 são quadrados perfeitos. Veja que 3x^2+ x= 4y^2+ y = 3(x^2- y^2) + (x-y)= y^2 = = (x-y)(3x+3y+1)=y^2 Seja d=mdc(x-y, 3x+3y+1). Suponha d1. Então existe p primo ; p|d. Então p|y^2 - p|y - p|(x-y) + y= x -

[obm-l] Re: [obm-l] alguém sabe?

2002-04-20 Por tôpico yurigomes
Na verdade, é possível provar que {x0/ x^x^x^x... converge}= [e^(-e), e^(1/e)] -- Mensagem original -- Olá Rui, Meu amigo Artur me apresentou esse problema na semana passada: Para x e^(1/e), temos x=e^(1/e+y), onde y 0 logo x^x = e^((1/e+y)*e^(1/e+y)) e^(e^(1/e+y-1)+y) , pois

Equação

2001-12-28 Por tôpico yurigomes
Olah a todos. Alguém consegue, por favor, resolver o problema abaixo!!! Ache todos x, y inteiros tais que 2x^4+ 1= y^2 []'s, Yuri ICQ: 64992515 -- Use o melhor sistema de busca da Internet Radar UOL - http://www.radaruol.com.br

Re: Um quadrado repartido

2001-11-28 Por tôpico yurigomes
| /| | x/ |1/2 | x/ | |--| | \ | |\ | | \| Espero que a figura satisfaça. Faça Pitágoras no triângulo retângulo da direita acima.Daí, x^2= (1/2)^2 + (1-x)^2 ...

Re: 4 Questoes

2001-11-26 Por tôpico yurigomes
4) Seja um paralelogramo ABCD. Traça-se uma reta que passa por D e corta o lado BC no ponto P e o prolongamento do lado AB no ponto Q. Se a área do triângulo DAC vale 8 e a área do quadrilátero ABCD vale 29, quanto vale a área do triângulo CPQ? Se a área de DAC=8, então a de ABCD será 16, e não

Ternas (x,y,z)

2001-11-21 Por tôpico yurigomes
Alguém pode resolver ou dar dicas para a seguinte questão??? Dado um inteiro positivo n, achar todas as ternas (x,y,z) de números reais tais que y*x^n + z*y^n + x*z^n = x*y^n + y*z^n + z*x^n []'s, Yuri ICQ: 64992515 -- Use o melhor sistema de busca

Re: Função

2001-10-29 Por tôpico yurigomes
Pensei um pouquinho, e tudo que consegui eh que f(x)= -c/x, x/=0, e f(x)=0 satisfaz, para c real. Serah que tem outra?? []'s, Yuri ICQ: 64992515 ___ http://www.zipmail.com.br O e-mail que vai aonde você está.

Re: Quadrados perfeitos...

2001-10-27 Por tôpico yurigomes
Basta ver que os quadrados perfeitos só podem acabar em 1,4,5,6 ou 9. Daih, a resposta eh letra e). []'s, Yuri ICQ: 64992515 ___ http://www.zipmail.com.br O e-mail que vai aonde você está.

Re: Problemas

2001-10-14 Por tôpico yurigomes
Para a questão 3, tente provar que o ponto de interseção de HM com a circunferência circunscrita a ABC pertencente ao arco BC é diametralmente oposto a A. -- Mensagem original -- Olá pessoal, Olhem só estes problemas: 1.Quantos triangulos diferentes ,de lados inteiros,podem ser

Re: Inglaterra-00-Plana

2001-08-25 Por tôpico yurigomes
Uma outra solução para esta questão é a seguinte: PMN = NPQ e NMQ = NPQ ( ângulos segmentos ). Daí, RNQ = NPQ + NQP ( ângulo externo ) = RNQ = PMQ Mas RMQ compreende o mesmo arco que RNQ = MQ é bissetriz PMR []'s, Yuri ICQ: 64992515

Problema interessante

2001-08-12 Por tôpico yurigomes
Alguém poderia me ajudar no problema abaixo. Ainda naum saí do canto! Sejam a e b inteiros não-negativos tais que ab=c^2, onde c é inteiro. Prove que existe um número n e inteiros x1, x2,...,xn, y1, y2,..., yn tais que x1^2 + x2^2 + ... + xn^2= a , y1^2 + y2^2 + ... + yn^2 = b e

Afinal, como foi???

2001-07-10 Por tôpico yurigomes
Ei, como eh que vcs foram realmente Eu tou acabando de ver q tu fez uma questão toda. Naum tou entendendo como eh que foram as pontuações. Tu jah eh menção né??? []'s, Yuri ICQ: 64992515 ___ http://www.zipmail.com.br O e-mail que

Correção

2001-07-10 Por tôpico yurigomes
O email Afinal, como foi? não era endereçado para a lista. []'s, Yuri ICQ: 64992515 ___ http://www.zipmail.com.br O e-mail que vai aonde você está.

Re: Problemas ....

2001-06-24 Por tôpico yurigomes
-- Mensagem original -- Caros colegas As medidas dos lados de um triângulo são x^2 + x + 1, 2x + 1 e x^2 - 1. Determine o maior lado e prove que o triângulo tem um ângulo de 120°. Um abraço YCM! Vejamos... Como x^2 - 1 é lado, temos x 1. Daí x^2 + x + 1 2x + 1 sss x^2 x sss

Problema de Análise

2001-02-28 Por tôpico yurigomes
Abaixo vai um problema de Anlise bem interessante: Dada f:[a,b)-R, suponha que f derivvel em (a,b) e tenha derivada limitada. Prove ento que f limitada. Aguardo a soluo. []'s, Yuri ___ http://www.zipmail.com.br O e-mail